Intégrale du mercredi 21 septembre

etanche
Modifié (September 2022) dans Analyse
Bonjour
Calculer $$\int_{0}^{+\infty} \frac{\ln(x)}{x^4+x^2+1}dx.$$
Merci

Réponses

  • $-\dfrac{\pi^2}{12}$.
  • L2M
    L2M
    Modifié (September 2022)
    Donc $-\zeta(2)/2$.
  • Numériquement c'est clair.
  • bd2017
    Modifié (September 2022)
    Bonjour
    $I=\displaystyle \int_0^\infty \dfrac{ \ln (x)}{x^4 + x^2 + 1} dx=\int_0^1 \dfrac{ \ln (x)}{x^4 + x^2 + 1} dx +\int_1^\infty  \dfrac{ \ln (x)}{x^4 + x^2 + 1} dx= \int_0^1  \dfrac{\left(1-x^2\right) \ln (x)}{x^4+x^2+1}  dx$
     (après avoir fait  le changement de variable $x\mapsto 1/x$ dans la deuxième intégrale) .  
    $I= \displaystyle \int_0 ^1  \dfrac{1}{1-x} \ln (x) dx$  $  \displaystyle-\int_0 ^1\dfrac{x}{1-x^2}\ln (x) dx -  3\displaystyle \int _0 ^1 \dfrac{ x^2}{1-x^6}  \ln (x) dx    $
    $I=I_1-I_2-3  I_3$ 
    Dans $I_3$  on fait le changement de variable  $x \mapsto x^{1/3}$
    $I_3 =\dfrac{1}{9}  \displaystyle \int_0^1 \dfrac{\ln(x)}{ \left(1-x^2\right)} dx =\dfrac{1}{ 9} I_4$  
    $I_4 = \displaystyle\int_0 ^1 (\dfrac{\ln (x)}{1-x}-\dfrac{x \ln (x)}{1-x^2}) dx =I_1- I_2$
    D'où $I= \dfrac{2}{3}( I_1-I_2)$
    Dans $I_2$  on fait le changment de variable $x\mapsto \sqrt{x}$  pour obtenir $I_2=\dfrac{1}{4} I_1$
    Finalement $I= \dfrac{I_1}{2} =-\dfrac{\pi ^2}{12} $

    Le calcul de $I_1$ étant bien connu













     
  • L2M
    L2M
    Modifié (September 2022)
    Je ne suis pas devant mon pc.

  • Boécien
    Modifié (September 2022)
    Le calcul de bd2017 m'a poussé à regarder $\int_{0}^{1}\frac{\log t}{1-t^{4}}dt$ Edit: Ce serait amusant de recenser des intégrales "non triviales"  $\int_{0}^{1}f(t)dt=0$ avec $f$ pas trop moche.


  • ManuTLS
    Modifié (September 2022)
    [Inutile de recopier un message présent sur le forum. Un lien suffit. AD]
    Euh.... ça va un peu trop vite pour moi.
    Est-ce qu'une bonne âme pourrait détailler un peu plus les calculs ?
  • Fin de partie
    Modifié (September 2022)
    @ManuTLS: Le calcul de Bd2017 vise à exprimer l'intégrale initiale en fonction de l'intégrale $\displaystyle \int_0^1 \frac{\ln x}{1-x}dx$
    On utilise le fait que $\displaystyle 1+x^2+x^4=\dfrac{1-x^6}{1-x^2}$
    On utilise aussi le fait que $d(x^3)=3x^2dx,\ d(x^2)=2xdx$
  • Fin de partie
    Modifié (September 2022)
    En suivant une autre voie, je suis arrivé à exprimer l'intégrale initiale sous la forme: $\displaystyle \frac{1}{2}\int_0^\infty  \ln\left(\frac{2\cosh z-1}{2\cosh z+1}\right)dz$
    PS: les détails du calcul:
    \begin{align}J&=\int_{0}^{+\infty} \frac{\ln x }{x^4+x^2+1}dx\\
    &\overset{u=\frac{1}{x}}=-\int_{0}^{+\infty} \frac{u^2\ln u}{u^4+u^2+1}du\\
    &=\frac{1}{2}\int_{0}^{+\infty} \frac{(1-x^2)\ln x }{x^4+x^2+1}dx\\
    &=\frac{1}{2}\int_{0}^{+\infty} \frac{\left(\frac{1}{x^2}-1\right)\ln x}{x^2+\frac{1}{x^2}+1}dx\\
    &=\frac{1}{2}\int_{0}^{1} \frac{\left(\frac{1}{x^2}-1\right)\ln x}{x^2+\frac{1}{x^2}+1}dx+\frac{1}{2}\underbrace{\int_{1}^{+\infty} \frac{\left(\frac{1}{x^2}-1\right)\ln x}{x^2+\frac{1}{x^2}+1}dx}_{u=\frac{1}{x}}\\
    &=\int_{0}^{1} \frac{\left(\frac{1}{x^2}-1\right)\ln x}{x^2+\frac{1}{x^2}+1}dx\\
    &\overset{u=\frac{1}{2}\left(x+\frac{1}{x}\right)}=\frac{1}{2}\int_1^\infty \frac{\ln\left(u-\sqrt{u^2-1}\right)}{u^2-\frac{1}{4}}du\\
    &=-\frac{1}{2}\int_1^\infty \frac{\ln\left(u+\sqrt{u^2-1}\right)}{u^2-\frac{1}{4}}du\\
    &=-\frac{1}{2}\int_1^\infty \frac{\text{arccosh }u}{u^2-\frac{1}{4}}du\\
    &\overset{z=\text{arccosh }u}=-\frac{1}{2}\int_0^\infty \frac{z\sinh z}{\cosh^2 z-\frac{1}{4}}dz\\
    &\overset{\text{IPP}}=-\frac{1}{2}\underbrace{\left[z\ln\left(\frac{2\cosh z-1}{2\cosh z+1}\right)\right]_0^\infty}_{=0}+\frac{1}{2}\int_0^\infty  \ln\left(\frac{2\cosh z-1}{2\cosh z+1}\right)dz\\
    \end{align}
  • Bonjour @jandri Montre tes cartes  :blush:
    Le 😄 Farceur


  • ManuTLS
    Modifié (September 2022)
    [Inutile de recopier l'avant-dernier message. Un lien suffit. AD]
    Merci, je vais me plonger dans tes explications et ramer un peu je pense :-)
  • jandri
    Modifié (September 2022)
    J'ai débuté comme bd2017 : $I=\displaystyle \int_0^1  \dfrac{\left(1-x^2\right) \ln (x)}{x^4+x^2+1}  dx$

    Puis $I=\displaystyle\int_0^1  \dfrac{\left(1-x^2\right)^2 \ln (x)}{1-x^6}  dx=\sum_{n=0}^{+\infty}(I_{6n}-2I_{6n+2}+I_{6n+4})$  avec $I_k=\displaystyle\int_0^1x^k\ln xdx=-\dfrac1{(k+1)^2}$.

    Ensuite $I=-\displaystyle\sum_{n=0}^{+\infty}\dfrac1{(6n+1)^2}+2\displaystyle\sum_{n=0}^{+\infty}\dfrac1{(6n+3)^2}-\displaystyle\sum_{n=0}^{+\infty}\dfrac1{(6n+5)^2}=-\zeta(2)+3\displaystyle\sum_{n=0}^{+\infty}\dfrac1{(6n+3)^2}+\displaystyle\sum_{n=0}^{+\infty}\dfrac1{(2n)^2}$.

    Puis $I=-\zeta(2)+\dfrac13\left(\zeta(2)-\dfrac14\zeta(2)\right)+\dfrac14\zeta(2)=-\dfrac12\zeta(2)$.
  • @Jandri: c'est le pendant série du calcul de Bd2017.
  • Merci Jandri, très astucieux 
    Le 😄 Farceur


  • Fin de partie
    Modifié (September 2022)
    Je continue ma promenade.
    On peut montrer aussi que l'intégrale initiale est égale à $\displaystyle \frac{1}{2}\int_0^{\frac{\pi}{2}} \frac{\ln\left(\frac{2-\cos x}{2+\cos x}\right)}{\cos x}dx$
    En factorisant $2$ dans le logarithme, en développant en série de Taylor  entière le logarithme, et en utilisant les valeurs des intégrales de Wallis je pressens qu'on peut terminer ce calcul (mais c'est moche de mon point de vue).
  • Je vous propose une belle généralisation : calculer $I_n=\displaystyle \int_0^\infty \dfrac{ \ln (x)}{1+x^2 +x^4+ \dots + x^{2n-2}} dx$ pour $n\geq2$.

    J'ai trouvé (avec l'aide de wolfram alpha) que le résultat s'exprime très simplement en fonction de $n$ mais je ne l'ai pas encore démontré pour tout $n$, je l'ai seulement vérifié par un calcul approché sur des valeurs de $n$ prises au hasard.
  • Fin de partie
    Modifié (September 2022)
    \begin{align}\frac{1}{2}\int_0^{\frac{\pi}{2}} \frac{\ln\left(\frac{2-\cos x}{2+\cos x}\right)}{\cos x}dx&=-\int_0^{\frac{\pi}{2}}\left(\sum_{n=0}^\infty \frac{\cos^{2n} x}{(2n+1)2^{2n+1}}\right)dx\\ &=-\sum_{n=0}^\infty \frac{1}{(2n+1)2^{2n+1}}\left(\int_0^{\frac{\pi}{2}} \cos^{2n} xdx\right)\\ &=-\sum_{n=0}^\infty \left(\frac{1}{(2n+1)2^{2n+1}}\times \frac{\pi}{2}\times \frac{\binom{2n}{n}}{2^{2n}}\right)\\ &=-\pi\sum_{n=0}^\infty \frac{\binom{2n}{n}}{(2n+1)4^{2n+1}} \end{align}
    Et on doit pouvoir transformer cette série en intégrale à coup de fonction Bêta. (mais la méthode reste laide)
  • bd2017
    Modifié (September 2022)
    Bonjour
    Pour la généralisation  de @jandri
    $I_n=-\dfrac{\pi ^2}  {4 (n)^2 } [ \dfrac{1}{ \sin ^2 (\frac{\pi }{2 (n)})} - \dfrac{1}{ \sin ^2 (\frac{3\pi }{2 (n)})}] $
     
  • L2M
    L2M
    Modifié (September 2022)
    Cette preuve consiste à utiliser la définition et quelques propriétés de la fonction zêta de Hurwitz \[\zeta(z,a)=\frac{1}{\Gamma(z)}\int_{0}^{+\infty} \frac{t^{z-1}e^{-at}}{1-e^{-t}}dt.\] Nous avons
    \begin{align*}I&=\int_{0}^{+\infty} \frac{\ln(x)}{x^4+x^2+1}dx\\&=\int_{-\infty}^{+\infty} \frac{te^{t}}{e^{4t}+e^{2t}+1}dt \ ;\qquad t=\ln x \quad \\&=\int_{0}^{+\infty} \frac{te^{t}}{e^{4t}+e^{2t}+1}dt + \int_{-\infty}^{0} \frac{te^{t}}{e^{4t}+e^{2t}+1}dt \\&=\int_{0}^{+\infty} \frac{te^{t}}{e^{4t}+e^{2t}+1}dt - \int_{0}^{+\infty} \frac{te^{3t}}{e^{4t}+e^{2t}+1}dt \ ;\qquad t:=-t \ \text{ dans la deuxième intégrale} \quad \\&=\int_{0}^{+\infty} \frac{te^{t}(1-e^{2t})}{e^{4t}+e^{2t}+1}dt \\&=\int_{0}^{+\infty} \frac{te^{t}(1-e^{2t})^2}{1-e^{6t}}dt \\&=\int_{0}^{+\infty} \frac{t(e^{t}-2e^{3t}+e^{5t})}{1-e^{6t}}dt \\&=\frac{1}{36}\int_{0}^{+\infty} \frac{t(e^{t/6}-2e^{t/2}+e^{5t/6})}{1-e^{t}}dt \ ;\qquad t:=6t \quad \\&=-\frac{1}{36}\int_{0}^{+\infty} \frac{t(e^{-5t/6}-2e^{-t/2}+e^{-t/6})}{1-e^{-t}}dt \\&=-\frac{1}{36}\left(\zeta(2,1-1/6) -2\zeta(2,1/2) +\zeta(2,1/6)\right) \end{align*} D'autre par, nous avons les relations \[ \zeta(2,a)+\zeta(2,1-a)=\frac{\pi^2}{\sin^2(a\pi)}\]
    et \[ \zeta(z,1/2)=(2^z-1)\zeta(z).\] Ainsi,\[I=-\frac{1}{36}\left(\frac{\pi^2}{\sin^2(\pi/6)}-6\zeta(2)\right)=-\frac{\pi^2}{12}.\]
  • Fin de partie
    Modifié (September 2022)
    Une intégrale proche. (traitée par le calcul des résidus)
    Je pense que la méthode utilisée par Jandri plus haut devrait fonctionner si on utilise les propriétés de la fonction digamma.
  • @Fin de partie c'est fait, j'ai corrigé le décalage
     
  • bd2017 : il faut aussi corriger le signe, ta formule donne $\pi^2/12$ pour $n=3$.

    Cette formule se généralise encore en posant $\alpha=2n$ :
    $\displaystyle \int_0^\infty \dfrac{ (1-x^2)\ln (x)}{1-x^{\alpha}} dx=\dfrac{\pi ^2}  {\alpha^2 } \left( \dfrac{1}{ \sin ^2 (3\pi /\alpha)} - \dfrac{1}{ \sin ^2 (\pi /\alpha)}\right)$ pour tout réel $\alpha>3$.

    Avec $t=x^\alpha$ et $\beta=1/\alpha$ c'est équivalent à : $\displaystyle \int_0^\infty \dfrac{ (t^{\beta-1}-t^{3\beta-1})\ln (t)}{1-t} dt=\pi ^2 \left( \dfrac{1}{ \sin ^2 (3\pi \beta)} - \dfrac{1}{ \sin ^2 (\pi \beta)}\right)$ pour tout réel $0<\beta<1/3$.

    En coupant l'intégrale en deux et en se ramenant à $[0,1]$ c'est une conséquence de l'égalité (valable pour $0<x<1$) :
    $\displaystyle \int_0^1\dfrac{ (t^{x-1}+t^{-x})\ln (t)}{1-t} dt=  -\dfrac{\pi ^2}{ \sin ^2 (\pi x)}$ qui s'obtient en dérivant $\displaystyle \int_0^1\dfrac{ t^{x-1}-t^{-x}}{1-t} dt=  \pi \;\rm{cot}(\pi x)$ qui s'écrit encore $\pi\; \rm{cot}(\pi x)=\displaystyle\sum_{n\in\Z}\dfrac1{x+n}$ (égalité bien connue).
    Cela démontre donc la formule annoncée.
  • L2M
    L2M
    Modifié (September 2022)
    Pour le cas général proposé par @jandri et pour simplifier, on peut remplacer $2n-2$ par $2n$ où $n \geq 1$. \begin{align*}I_n&=\int_{0}^{+\infty} \frac{\ln(x)}{x^{2n}+...+x^2+1}dx\\&=\int_{-\infty}^{+\infty} \frac{te^{t}}{e^{2nt}+...+e^{2t}+1}dt \ ;\qquad t=\ln x \quad \\&=\int_{0}^{+\infty} \frac{te^{t}}{e^{2nt}+...+e^{2t}+1}dt + \int_{-\infty}^{0} \frac{te^{t}}{e^{2nt}+...+e^{2t}+1}dt \\&=\int_{0}^{+\infty} \frac{te^{t}}{e^{2nt}+...+e^{2t}+1}dt - \int_{0}^{+\infty} \frac{te^{(2n-1)t}}{e^{2nt}+...+e^{2t}+1}dt \ ;\qquad t:=-t \ \text{ dans la deuxième intégrale} \quad \\&=\int_{0}^{+\infty} \frac{te^{t}(1-e^{2(n-1)t})}{e^{2nt}+...+e^{2t}+1}dt \\&=\int_{0}^{+\infty} \frac{te^{t}(1-e^{2(n-1)t})(1-e^{2t})}{1-e^{2(n+1)t}}dt \\&=\int_{0}^{+\infty} \frac{t(e^{t}-e^{3t}-e^{(2n-1)t}+e^{(2n+1)t})}{1-e^{2(n+1)t}}dt \\&=\frac{1}{4(n+1)^2}\int_{0}^{+\infty} \frac{t(e^{t/(2n+2)}-e^{3t/(2n+2)}-e^{(2n-1)t/(2n+2)}+e^{(2n+1)t/(2n+2)})}{1-e^{t}}dt \ ;\qquad t:=2(n+1)t \quad \\&=-\frac{1}{4(n+1)^2}\int_{0}^{+\infty} \frac{t(e^{-(1-1/(2n+2))t}-e^{-(1-3/(2n+2))t}-e^{-3t/(2n+2)}+e^{-t/(2n+2)})}{1-e^{-t}}dt \\&=-\frac{1}{4(n+1)^2}\left(\zeta(2,1-1/(2n+2)) -\zeta(2,1-3/(2n+2))-\zeta(2,3/(2n+2))+\zeta(2,1/(2n+2))\right). \end{align*} D'après la relation \[\zeta(2,a)+\zeta(2,1-a)=\frac{\pi^2}{\sin^2(a\pi)},\]\[I_n=\frac{\pi^2}{4(n+1)^2}\left(\frac{1}{\sin^2(3\pi/(2n+2))}-\frac{1}{\sin^2(\pi/(2n+2))}\right).\]
  • ManuTLS
    Modifié (September 2022)
    @Fin de partie, en utilsant (une partie) de tes indications, j'arrive à ça :$$I=\int_{0}^{1}\frac{\ln(x)}{1-x^{6}}dx-\int_{0}^{1}\frac{2x^{2}\ln(x)}{1-x^{6}}dx+\int_{0}^{1}\frac{x^{4}\ln(x)}{1-x^{6}}dx.$$Mais après... je ne vois pas comment arriver à l’expression de @bd2017.
    Bon, je me remets lentement aux maths, il faut être indulgent avec moi.
  • Fin de partie
    Modifié (September 2022)
    On s'en fiche comment il a obtenu cette décomposition tout ce qui importe est que:
    \begin{align}\frac{1}{1-x}-\frac{x}{1-x^2}-\frac{3x^2}{1-(x^3)^2}=\frac{1-x^2}{1+x^2+x^4}\end{align}
  • Fin de partie
    Modifié (September 2022)
    \begin{align}\int_0^1 \frac{(1-x^2)\ln x}{1+x^2+x^4}dx=\int_0^1\frac{\ln x}{1-x}dx-\underbrace{\int_0^1\frac{x\ln x}{1-x^2}dx}_{u=x^2}-\underbrace{\int_0^1\frac{3x^2\ln x}{1-(x^3)^2}dx}_{u=x^3}\end{align}
  • Pour le coup, c'est vraiment le genre d'intégrale qui est faite pour être calculé par résidus. Il faut considérer la fonction complexe auxiliaire $$\frac{(\ln(z))^2}{z^4+z^2+1}$$ où la détermination du logarithme complexe a pour coupure $[0,+\infty[$. Ensuite on intègre sur un Pacman qui "mange" $[0,+\infty[$.
  • Pourquoi le logarithme au carré ?
  • @Fin de partie le changement de variable n'est pas le problème, c'est plus la décomposition. En effet, même si l'égalité se vérifie après quelques lignes de calcul, comment "voir" ça, y penser? C'est une décomposition "connue" ou "classique" ? La vérifier est une chose, mais la trouver est moins évident pour moi....
  • @ManuTLS: Pour ce genre de calculs, parfois les méthodes bien connues fonctionnent, parfois il faut être plus créatif. C'est ça tout l'intérêt, selon moi.
    Ici, cela fonctionne car $1-x^6$ se factorise bien.
  • @Math Cross : Teste avec le logarithme seul et tu comprendras vite.  :p

    De manière générale quand on utilise cette technique par résidu, il faut à chaque fois augmenter le degré d'une unité. 
  • L2M
    L'égalité que j'utilise, $\displaystyle \int_0^1\dfrac{ (t^{x-1}+t^{-x})\ln (t)}{1-t} dt=  -\dfrac{\pi ^2}{ \sin ^2 (\pi x)}$, devient en posant $t=e^{-u}$ :  
    $\displaystyle\zeta(2,x)+\zeta(2,1-x)=\frac{\pi^2}{\sin^2(x\pi)}$
Connectez-vous ou Inscrivez-vous pour répondre.